Đến nội dung

An Infinitesimal nội dung

Có 155 mục bởi An Infinitesimal (Tìm giới hạn từ 25-04-2020)



Sắp theo                Sắp xếp  

#716765 $\lim_{x\rightarrow 0} (1+x^2)^{cotg(x)^2}...

Đã gửi bởi An Infinitesimal on 20-10-2018 - 23:01 trong Dãy số - Giới hạn

$\lim_{x\rightarrow 0} (1+x^2)^{\cot^2 x}.$

 

 

Ta có $$(1+x^2)^{\cot^2 x} = e^{\cot^2 x \ln{(1+x^2)}}.$$

 

Hơn nữa, 

$$\lim_{x\to0}\cot^2 x \ln{(1+x^2)}= \lim_{x\to0}\cos^2 x \frac{\ln{(1+x^2)}}{x^2}\frac{1}{\frac{\sin^2x}{x^2}}=1. $$

 

Suy ra $$\lim_{x\to0} (1+x^2)^{\cot^2 x} =e.$$




#716688 Chứng minh dãy số sau có giới hạn $L$

Đã gửi bởi An Infinitesimal on 18-10-2018 - 06:42 trong Dãy số - Giới hạn

Sao Un-U(n-1) lại <0 ạ. Em chưa hiểu lắm

 

 

Nếu không có giả thiết $a>0$ thì đề sai.

 

Khi $a>0,$ dùng BĐT Cauchy, suy ra $u_n\ge \sqrt{x} \forall n\ge 1.$ 

 

Nếu không có giả thiết $a>0$ thì đề sai.

 

Khi $a>0,$ dùng BĐT Cauchy, suy ra $u_n\ge \sqrt{x} \forall n\ge 1.$ 

 



#716639 Chứng minh dãy số sau có giới hạn $L$

Đã gửi bởi An Infinitesimal on 16-10-2018 - 18:50 trong Dãy số - Giới hạn

dxuhivlrwf9g2f4qd.png
Giả sử √x=L.
Anh chị giúp e chứng minh dãy trên là hữu hạn và có giới hạn là L với.
Em rất cảm ơn ạ :)

 

Nếu không có giả thiết $a>0$ thì đề sai.

 

Khi $a>0,$ dùng BĐT Cauchy, suy ra $u_n\ge \sqrt{x} \forall n\ge 1.$ Hơn nữa, $u_{n}-u_{n-1}=\frac{x-u_{n-1}^2}{2u_{n-1}}\le 0 \forall n\ge 2.$
Vì thế $\left\{ u_n\right\}_{n\ge 2}$ là dãy giảm và bị chặn dưới. Do đó, dãy này hội tụ. Gọi $b= \lim u_n, b\ge \sqrt{x}.$

 

Cho hệ thức truy hồi qua giới hạn, ta nhận được phương trình: $b= \frac{1}{2}\left( b+\frac{x}{b}\right).$ Suy ra $b=L=\sqrt{x}.$ Điều cần phải chứng minh.




#716520 Tính liên tục của đạo hàm

Đã gửi bởi An Infinitesimal on 13-10-2018 - 05:59 trong Giải tích

Nếu hàm $f$ liên tục trên $[a,b]$ và khả vi trên $(a,b$) thì $f'$ có liên tục trên $[a,b]$ không?

 

Liệu $f$ có khả vi tại $a$ hay không mà... $f '$ liên tục tại $a$?




#716439 Giới hạn hàm nhiều biến

Đã gửi bởi An Infinitesimal on 09-10-2018 - 18:56 trong Giải tích

Chào các anh chị, anh chị có thể giúp em giải các bài tập sau được không ạ, em cảm ơn nhiều ạ

lim (x^2+y^2) e^-(x+y) khi (x,y) -> (+vô cùng, +vô cùng)

 

Dùng BĐT $e^u \ge \frac{u^3}{3!}$ với $u>0.$




#716226 Cho $x\to +\infty$. Chứng minh rằng: $\frac...

Đã gửi bởi An Infinitesimal on 02-10-2018 - 07:38 trong Giải tích

Cho $x\to +\infty$. Chứng minh rằng: $\frac{arctan(x)}{1+x^2}=O(\frac{1}{x^2})$

 

Điều này dễ thấy vì $\lim_{x\to \infty} \arctan x=\frac{\pi}{2}.$




#715972 Cho $0\leq U_n \leq 2$ va $U_{n}+U_{n...

Đã gửi bởi An Infinitesimal on 24-09-2018 - 17:50 trong Dãy số - Giới hạn

 

$0\leq U_n \leq 2$ va $U_{n}+U_{n+2}\geq 2U_{n+1}$ , $n \leq 1$
cmr $0 \leq n(U_n-U_{n+1}) \leq 2$ 

 

 

Vì dãy $\left\{u_k-u_{k+1} \right\}$ là dãy giảm nên 

$$n(u_{n}-u_{n+1})\le (u_1-u_2)+(u_2-u_3)+...+(u_n-u_{n+1})=u_1-u_{n+1}\le 2.$$




#715688 $x_{n+1}=\frac{x_{n}+y_{n}}...

Đã gửi bởi An Infinitesimal on 18-09-2018 - 13:10 trong Dãy số - Giới hạn

Cho $x_{1}=a,y_{1}=b;
và x_{n+1}=\frac{x_{n}+y_{n}}{2};
y_{n+1}=\sqrt{x_{n}y_{n}}$
Tìm lim $x_{n},y_{n}$

Sao bạn không đề cập đến điều kiện của $a$ và $b$?




#715370 $\sum_{n=1}^{+\infty}{\frac...

Đã gửi bởi An Infinitesimal on 10-09-2018 - 02:11 trong Giải tích

Kết hợp sao vậy bạn? Giải thích rõ hơn dùm mình nha! Cảm ơn!

 

1) Khảo sát sự hội tụ của chuỗi số sau:

                    $\sum_{n=1}^{+\infty}{\frac{\cos{n}}{n}}.$

2) Dùng tiêu chuẩn Cauchy xét tính hội tụ của dãy số sau:

$S_n=\frac{\cos{1^n}}{2^1}+\frac{\cos{2^n}}{2^2}+...+\frac{\cos{n^n}}{2^n}.$

 

$T_n=\frac{|\cos{1^n}|}{2^1}+\frac{|\cos{2^n}|}{2^2}+...+\frac{|\cos{n^n}|}{2^n}$. Dãy $\left\{ T_n\right\}$ hội tụ vì dãy này tăng và bị chặn trên bởi 1. 
Tìm ra chặn trên của dãy nhờ đánh giá $\frac{1}{2}+\frac{1}{2^2}+...+\frac{1}{2^n}<1.$

Suy ra $\left\{ S_n\right\}$ hội tụ.




#715369 Chứng minh rằng: $n!>(\frac{n}{3})^...

Đã gửi bởi An Infinitesimal on 10-09-2018 - 02:05 trong Dãy số - Giới hạn

có đơn giản quá không nhỉ

 

Ý bạn là thế nào?

 

$e=\lim_{x\rightarrow \infty }(1+\frac{1}{x})^x$

 

Vậy làm sao định nghĩa số $(1+\frac{1}{x})^x$ khi $x$ là số vô tỷ?




#715368 Chứng minh dãy Cauchy thì hội tụ

Đã gửi bởi An Infinitesimal on 10-09-2018 - 01:56 trong Dãy số - Giới hạn

Có vẻ như anh nhầm gì đó ở chỗ chú ý. Theo nội dung tiêu chuẩn Cauchy, dãy số $u_n$ hội tụ khi và chỉ khi nó là dãy Cauchy.

 

Bạn đọc không kỹ. Dãy thì chưa chắc là dãy số.




#714695 Cho dãy số (un):

Đã gửi bởi An Infinitesimal on 22-08-2018 - 22:57 trong Dãy số - Giới hạn

"Ngịch đảo" ta sẽ dẫn ra dãy truy hồi tuyến tính $2v_{n+2}=v_{n+1}+v_{n}, n\ge 1,$ trong đó $v_n= \frac{1}{u_n}.$

 

Giải tìm $v_n$ theo $n$. Từ đó, ta xác định được $\lim u_n.$




#714422 Tính tích phân suy rộng $\int_{1}^{\infty...

Đã gửi bởi An Infinitesimal on 15-08-2018 - 17:21 trong Giải tích

Tính tích phân suy rộng $\int_{1}^{\infty } \frac{ln(1+x)}{x} dx$ 

 

Sử dụng BĐT $\frac{\ln{(1+x)}}{x}\ge \frac{\ln 2}{x}>0 \, \forall x\ge 1$ để chứng minh TPSR phân kỳ.




#714421 $n_{n+1}=\sqrt{u_n} +\sqrt{u_{n-...

Đã gửi bởi An Infinitesimal on 15-08-2018 - 17:19 trong Dãy số - Giới hạn

Cho dãy số $(u_n)$ xác định bởi:

$\left\{\begin{matrix}u_1=1, u_2=2 \\ u_{n+1}=\sqrt{u_n} +\sqrt{u_{n-1}} , \forall n\geq 2 \end{matrix}\right.$

CMR: $(u_n)$ tăng và bị chặn trên.

 

(+) Dãy bị chặn trên bởi 4.

(+) Dãy tăng được chứng minh bằng qui nạp.




#713782 Chứng minh rằng: $n!>(\frac{n}{3})^...

Đã gửi bởi An Infinitesimal on 03-08-2018 - 17:58 trong Dãy số - Giới hạn

Có lẽ bài toán của mình chính là như kiểu đang chứng minh dãy $\left\{\left(1+\frac{1}{n}\right)^n\right\}$ là dãy tăng và hội tụ về $e.$

 

Mình đọc thấy có điều gì đó kỳ kỳ! Vậy e là gì?




#713654 Chứng minh rằng: $n!>(\frac{n}{3})^...

Đã gửi bởi An Infinitesimal on 01-08-2018 - 16:01 trong Dãy số - Giới hạn

 

 

Chứng minh rằng: $n!>(\frac{n}{3})^{n}$ với $\forall n\in\mathbb{N}^*$

 

Bất đẳng thức "mạnh hơn" là $n!>(\frac{n}{e})^{n} \forall n\ge 1.$

 

Đặt $u_n= \dfrac{n!}{\left(\frac{n}{e}\right)^n}.$

 

Ta có $\frac{u_{n+1}}{u_n}=\dfrac{e}{\left(1+\frac{1}{n}\right)^n}>1.$

 

Lưu ý: Khi định nghĩa, $e$, ta đã có dãy $\left\{\left(1+\frac{1}{n}\right)^n\right\}$ là dãy tăng và hội tụ về $e.$

 

Hơn nữa, $u_1>1$ nên $u_n>1 \forall n\ge 1.$ Suy ra điều phải chứng minh.




#713355 Cực trị của hàm 2 biến

Đã gửi bởi An Infinitesimal on 27-07-2018 - 22:21 trong Giải tích

Tìm cực trị của hàm $z = x^3y^2(6 - x - y)$, $x>0$, $y>0$.

 

Trên miền $x>0, y>0$, hàm số chỉ có duy nhất một điểm dừng $(3,2)$ và điểm dừng đó là điểm cực tiểu.




#713162 $\displaystyle \lim_{x\to 0}(\displaystyle...

Đã gửi bởi An Infinitesimal on 24-07-2018 - 18:42 trong Giải tích

Mình nhìn hơi vội! Để mình xử lý lại!

 

 

 

 

 

Nháp: 

Ta có 

$A = \begin{bmatrix} 1 & \dfrac{x}{n}\\ -\dfrac{x}{n} & 1 \end{bmatrix}= E+\frac{1}{n}B,$ trong đó $B= \begin{bmatrix} 0 & x\\ -x & 0 \end{bmatrix}$

$A^n-E=B+ \sum_{k=2}^n\frac{C_n^k}{n^k}B^k.$

 


Sao lại tiến về ma trận không?

 

Mình nhìn hơi vội! Để mình xử lý lại!

 

 

 

 

 

Nháp: 

Ta có 

$A = \begin{bmatrix} 1 & \dfrac{x}{n}\\ -\dfrac{x}{n} & 1 \end{bmatrix}= E+\frac{1}{n}B,$ trong đó $B= \begin{bmatrix} 0 & x\\ -x & 0 \end{bmatrix}$

$A^n-E= \sum_{k=1}^n\frac{C_n^k}{n^k}B^k.$

Đặt $S_n= E+\sum_{k=1}^n\frac{1}{k!}B^k.$

 

 

Nhận xét:

1) Dãy $\left\{S_n\right\}$ hội tụ về $S:=e^{B}.$

 

2) Dãy $\left\{A^n-S_n\right\}$ hội tụ về  $0.$

 

(Cần kiểm tra 2.)

 

$A^n-S_n= \sum_{k=1}^n\frac{1}{k!}\left(\displaystyle\prod_{i=1}^{n-k}(1-\frac{k}{n})-1\right)B^k.$




#713053 ${{u}_{n+1}}=\dfrac{u_{n...

Đã gửi bởi An Infinitesimal on 22-07-2018 - 21:12 trong Dãy số - Giới hạn

Mình đang muốn hướng tìm số hạng tổng quát. Liệu có tìm được không nhỉ?

 

Hãy cho mình một lý do để ta lại đi theo tiếp cận phức tạp hơn hướng đơn giản?




#713051 $\displaystyle \lim_{x\to 0}(\displaystyle...

Đã gửi bởi An Infinitesimal on 22-07-2018 - 21:04 trong Giải tích

Tìm

$\displaystyle \lim_{x\to 0}(\displaystyle \lim_{n\to \infty}(\dfrac{1}{x}(A^n-E)))$

trong đó $E$ là ma trận đơn vị

và $A = \begin{bmatrix} 1 & \dfrac{x}{n}\\ -\dfrac{x}{n} & 1 \end{bmatrix}, n \in \mathbb{N^*}$

 

Giới hạn bên trong tiến về ma trân không (ma trận vuông cấp 2). 




#712981 ${{u}_{n+1}}=\dfrac{u_{n...

Đã gửi bởi An Infinitesimal on 22-07-2018 - 02:31 trong Dãy số - Giới hạn

Cho dãy số $({{u}_{n}})$ xác định bởi ${{u}_{1}}>0$ và ${{u}_{n+1}}=\dfrac{u_{n}^{3}+4{{u}_{n}}}{u_{n}^{2}+1}$ với mọi $n\ge 1.$ Chứng minh rằng ${{u}_{n}}<\dfrac{2{{u}_{1}}+3(n-1)}{2}$ với mọi $n\ge 2.$

 

Mấu chốt là tính chất sau: $u_{n+1}\le u_n+\frac{3}{2}\, \forall n\ge 1.$




#712565 Tìm giới hạn dãy $1+\frac{1}{2} +...+\frac...

Đã gửi bởi An Infinitesimal on 15-07-2018 - 13:23 trong Dãy số - Giới hạn

Tìm giới hạn dãy $1+\frac{1}{2} +...+\frac{1}{n+1}$

P/s:Mong mọi người bỏ ra chút thời gian giúp mình với ạ!

 

Dùng đánh giá $\ln{(1+x)}\le x,$ ta có 

$\frac{1}{k}\ge \ln\left( 1+\frac{1}{k}\right)=\ln{(k+1)}-\ln k. $

Suy ra 

$$1+\frac{1}{2} +...+\frac{1}{n+1}\ge \ln{(n+1).}$$

Do đó, 

$$\lim_{n\to\infty} \left(1+\frac{1}{2} +...+\frac{1}{n+1}\right)=\infty.$$




#712564 Tìm $\lim u_n$ biết $u_n=\frac{1}{2...

Đã gửi bởi An Infinitesimal on 15-07-2018 - 13:18 trong Dãy số - Giới hạn

Tìm $\lim u_n$ biết $u_n=\frac{1}{2\sqrt{1}+1\sqrt{2}}+\frac{1}{3\sqrt{2}+2\sqrt{3}}+...+\frac{1}{(n+1)\sqrt{n}+n\sqrt{n+1}}$

 

Dùng đẳng thức $$\dfrac{1}{(k+1)\sqrt{k}+k\sqrt{k+1}}= \dfrac{1}{\sqrt{k}\sqrt{k+1}\left( \sqrt{k+1}+\sqrt{k}\right)}=\dfrac{\sqrt{k+1}-\sqrt{k}}{\sqrt{k}\sqrt{k+1}}= \frac{1}{\sqrt{k}}-\frac{1}{\sqrt{k+1}}.$$




#712563 Hàm số sau đây có bao nhiêu điểm gián đoạn $$f(x)=\begin{...

Đã gửi bởi An Infinitesimal on 15-07-2018 - 13:15 trong Dãy số - Giới hạn

Hàm số sau đây có bao nhiêu điểm gián đoạn
$$f(x)=\begin{cases}\frac{|x|}{x}, x \ne 0\\0, x=0\end{cases}$$

 

Dễ dàng kiểm tra hàm số này chỉ có duy nhất điểm gián đoạn: $x=0.$




#710342 cho dãy số thỏa u1=672

Đã gửi bởi An Infinitesimal on 09-06-2018 - 10:28 trong Dãy số - Giới hạn

ai giúp mình với

Nghịch đảo sẽ thấy dãy đơn giản theo dãy $\left\{\frac{1}{u_n}\right\}.$